Combinatorics question from IMO 2012What are the probabilities in this particular spin-off of BlackJackA Probability Question From Real LifePlay with pairs of numbersRelabelling players in a tournamentGame of cards and GCDA game with numbers from MEMO $2013$A game with stones and finding the winning setAlice and Bob number sum gameDecomposing into sum gameQuestion about gcd and divisbility, check part (b) and (c)

Valid Badminton Score?

Was the picture area of a CRT a parallelogram (instead of a true rectangle)?

How to be diplomatic in refusing to write code that breaches the privacy of our users

Was Spock the First Vulcan in Starfleet?

Do there exist finite commutative rings with identity that are not Bézout rings?

Is HostGator storing my password in plaintext?

Why are on-board computers allowed to change controls without notifying the pilots?

Print name if parameter passed to function

Coordinate position not precise

Applicability of Single Responsibility Principle

What is the intuitive meaning of having a linear relationship between the logs of two variables?

Mapping a list into a phase plot

Failed to fetch jessie backports repository

Products and sum of cubes in Fibonacci

Teaching indefinite integrals that require special-casing

Tiptoe or tiphoof? Adjusting words to better fit fantasy races

How could Frankenstein get the parts for his _second_ creature?

What will be the benefits of Brexit?

Generic lambda vs generic function give different behaviour

Why Were Madagascar and New Zealand Discovered So Late?

Your magic is very sketchy

What is the oldest known work of fiction?

Is it okay / does it make sense for another player to join a running game of Munchkin?

Short story about space worker geeks who zone out by 'listening' to radiation from stars



Combinatorics question from IMO 2012


What are the probabilities in this particular spin-off of BlackJackA Probability Question From Real LifePlay with pairs of numbersRelabelling players in a tournamentGame of cards and GCDA game with numbers from MEMO $2013$A game with stones and finding the winning setAlice and Bob number sum gameDecomposing into sum gameQuestion about gcd and divisbility, check part (b) and (c)













2












$begingroup$


This is the 3rd problem of the International Mathematical Olympiad 2012.




The liar’s guessing game is a game played between two players $A$ and $B$. The rules of the game depend on two positive integers $k$
and $n$ which are known to both players.
At the start of the game the player $A$
chooses integers $x$ and $N$
with $1leq xleq N$. Player $A$ keeps $x$ secretly, and truthfully tells $N$ to the player $B$. The player $B$ now tries to obtain information about $x$ by asking player $A$ questions as follows: each question consists of $B$ specifying an arbitrary set $S$
of positive integers (possibly one specified in some previous question), and asking $A$ whether $x$
belongs to $S$
Player $B$
may ask as many questions as he wishes. After each question, player $A$ must immediately answer it with yes or no, but is allowed to lie as many times as she wants; the only restriction is that, among any $k+1$ consecutive answers, at least one answer must be truthful.



After $B$ has asked as many questions as he wants, he must specify a set $X$ of at most $n$ positive integers. If $xin X$, then $B$ wins; otherwise, he loses. Prove that:



(a) If $𝑛geq2k$
then $B$ has a winning strategy.



(b) There exists a positive integer $𝑘_0$
such that for every $𝑘geq𝑘_0$
there exists an integer $𝑛geq1.99𝑘$
for which 𝐵 cannot guarante a win.




How to solve this question without using complex methods?.










share|cite|improve this question











$endgroup$











  • $begingroup$
    Learn $LaTeX$.
    $endgroup$
    – Rodrigo de Azevedo
    Mar 17 at 13:13






  • 1




    $begingroup$
    Can you solve it already using "complex methods" then? ;-)
    $endgroup$
    – drhab
    Mar 17 at 13:16















2












$begingroup$


This is the 3rd problem of the International Mathematical Olympiad 2012.




The liar’s guessing game is a game played between two players $A$ and $B$. The rules of the game depend on two positive integers $k$
and $n$ which are known to both players.
At the start of the game the player $A$
chooses integers $x$ and $N$
with $1leq xleq N$. Player $A$ keeps $x$ secretly, and truthfully tells $N$ to the player $B$. The player $B$ now tries to obtain information about $x$ by asking player $A$ questions as follows: each question consists of $B$ specifying an arbitrary set $S$
of positive integers (possibly one specified in some previous question), and asking $A$ whether $x$
belongs to $S$
Player $B$
may ask as many questions as he wishes. After each question, player $A$ must immediately answer it with yes or no, but is allowed to lie as many times as she wants; the only restriction is that, among any $k+1$ consecutive answers, at least one answer must be truthful.



After $B$ has asked as many questions as he wants, he must specify a set $X$ of at most $n$ positive integers. If $xin X$, then $B$ wins; otherwise, he loses. Prove that:



(a) If $𝑛geq2k$
then $B$ has a winning strategy.



(b) There exists a positive integer $𝑘_0$
such that for every $𝑘geq𝑘_0$
there exists an integer $𝑛geq1.99𝑘$
for which 𝐵 cannot guarante a win.




How to solve this question without using complex methods?.










share|cite|improve this question











$endgroup$











  • $begingroup$
    Learn $LaTeX$.
    $endgroup$
    – Rodrigo de Azevedo
    Mar 17 at 13:13






  • 1




    $begingroup$
    Can you solve it already using "complex methods" then? ;-)
    $endgroup$
    – drhab
    Mar 17 at 13:16













2












2








2


1



$begingroup$


This is the 3rd problem of the International Mathematical Olympiad 2012.




The liar’s guessing game is a game played between two players $A$ and $B$. The rules of the game depend on two positive integers $k$
and $n$ which are known to both players.
At the start of the game the player $A$
chooses integers $x$ and $N$
with $1leq xleq N$. Player $A$ keeps $x$ secretly, and truthfully tells $N$ to the player $B$. The player $B$ now tries to obtain information about $x$ by asking player $A$ questions as follows: each question consists of $B$ specifying an arbitrary set $S$
of positive integers (possibly one specified in some previous question), and asking $A$ whether $x$
belongs to $S$
Player $B$
may ask as many questions as he wishes. After each question, player $A$ must immediately answer it with yes or no, but is allowed to lie as many times as she wants; the only restriction is that, among any $k+1$ consecutive answers, at least one answer must be truthful.



After $B$ has asked as many questions as he wants, he must specify a set $X$ of at most $n$ positive integers. If $xin X$, then $B$ wins; otherwise, he loses. Prove that:



(a) If $𝑛geq2k$
then $B$ has a winning strategy.



(b) There exists a positive integer $𝑘_0$
such that for every $𝑘geq𝑘_0$
there exists an integer $𝑛geq1.99𝑘$
for which 𝐵 cannot guarante a win.




How to solve this question without using complex methods?.










share|cite|improve this question











$endgroup$




This is the 3rd problem of the International Mathematical Olympiad 2012.




The liar’s guessing game is a game played between two players $A$ and $B$. The rules of the game depend on two positive integers $k$
and $n$ which are known to both players.
At the start of the game the player $A$
chooses integers $x$ and $N$
with $1leq xleq N$. Player $A$ keeps $x$ secretly, and truthfully tells $N$ to the player $B$. The player $B$ now tries to obtain information about $x$ by asking player $A$ questions as follows: each question consists of $B$ specifying an arbitrary set $S$
of positive integers (possibly one specified in some previous question), and asking $A$ whether $x$
belongs to $S$
Player $B$
may ask as many questions as he wishes. After each question, player $A$ must immediately answer it with yes or no, but is allowed to lie as many times as she wants; the only restriction is that, among any $k+1$ consecutive answers, at least one answer must be truthful.



After $B$ has asked as many questions as he wants, he must specify a set $X$ of at most $n$ positive integers. If $xin X$, then $B$ wins; otherwise, he loses. Prove that:



(a) If $𝑛geq2k$
then $B$ has a winning strategy.



(b) There exists a positive integer $𝑘_0$
such that for every $𝑘geq𝑘_0$
there exists an integer $𝑛geq1.99𝑘$
for which 𝐵 cannot guarante a win.




How to solve this question without using complex methods?.







combinatorics contest-math






share|cite|improve this question















share|cite|improve this question













share|cite|improve this question




share|cite|improve this question








edited Mar 17 at 16:12









Dr. Mathva

2,892527




2,892527










asked Mar 17 at 13:08









Chand16Chand16

276




276











  • $begingroup$
    Learn $LaTeX$.
    $endgroup$
    – Rodrigo de Azevedo
    Mar 17 at 13:13






  • 1




    $begingroup$
    Can you solve it already using "complex methods" then? ;-)
    $endgroup$
    – drhab
    Mar 17 at 13:16
















  • $begingroup$
    Learn $LaTeX$.
    $endgroup$
    – Rodrigo de Azevedo
    Mar 17 at 13:13






  • 1




    $begingroup$
    Can you solve it already using "complex methods" then? ;-)
    $endgroup$
    – drhab
    Mar 17 at 13:16















$begingroup$
Learn $LaTeX$.
$endgroup$
– Rodrigo de Azevedo
Mar 17 at 13:13




$begingroup$
Learn $LaTeX$.
$endgroup$
– Rodrigo de Azevedo
Mar 17 at 13:13




1




1




$begingroup$
Can you solve it already using "complex methods" then? ;-)
$endgroup$
– drhab
Mar 17 at 13:16




$begingroup$
Can you solve it already using "complex methods" then? ;-)
$endgroup$
– drhab
Mar 17 at 13:16










1 Answer
1






active

oldest

votes


















1












$begingroup$

All but the youngest IMO problems have solutions on imomath.com. I've reproduced verbatim from here:




The game can be reformulated in an equivalent one: The player $A$
chooses an element $x$ from the set $S$ (with $|S|=N$) and the player
$B$ asks the sequence of questions. The $j$-th question consists of
$B$ choosing a set $D_jsubseteq S$ and player $A$ selecting a set
$P_jinleft Q_j,Q_j^Cright$. The player $A$ has to make sure
that for every $jgeq 1$ the following relation holds: $xin P_jcup
> P_j+1cupcdots cup P_j+k$
.



The player $B$ wins if after a finite number of steps he can choose a
set $X$ with $|X|leq n$ such that $xin X$.



(a) It suffices to prove that if $Ngeq 2^k+1$ then the player $B$ can
determine a set $S^primesubseteq S$ with $|S^prime|leq N-1$
such that $xin S^prime$.



Assume that $Ngeq 2^n+1$.



In the first move $B$ selects any set $D_1subseteq S$ such that
$|D_1|geq 2^k-1$ and $|D_1^C|geq 2^k-1$. After receiving the set
$P_1$ from $A$, $B$ makes the second move. The player $B$ selects a
set $D_2subseteq S$ such that $|D_2cap P_1^C|geq 2^k-2$ and
$|D_2^Ccap P_1^C|geq 2^k-2$. The player $B$ continues this way: in
the move $j$ he/she chooses a set $D_j$ such that $|D_jcap P_j^C|geq
> 2^k-j$
and $|D_j^Ccap P_j^C|geq 2^k-j$.



In this way the player $B$ has obtained the sets $P_1$, $P_2$,
$dots$, $P_k$ such that $left(P_1cup cdots cup P_kright)^Cgeq
> 1$
. Then $B$ chooses the set $D_k+1$ to be a singleton containing
any element outside of $P_1cupcdots cup P_k$. There are two cases
now:



  • The player $A$ selects $P_k+1=D_k+1^C$. Then $B$ can take $S^prime=Ssetminus D_k+1$ and the statement is proved.

  • The player $A$ selects $P_k+1=D_k+1$. Now the player $B$ repeats the previous procedure on the set $S_1=Ssetminus D_k+1$
    to obtain the sequence of sets $P_k+2$, $P_k+3$, $dots$,
    $P_2k+1$. The following inequality holds: $$left|S_1setminus
    > left(P_k+2cdots P_2k+1right)right|geq 1,$$

    since $|S_1|geq 2^k$. However, now we have $$left|left(P_k+1cup P_k+2cupcdotscup
    > P_2k+1right)^Cright|geq 1,$$
    and we may take
    $S^prime=P_k+1cup cdots cup P_2k+1$.

(b) Let $p$ and $q$ be two positive integers such that $1.99< p< q<
> 2$
. Let us choose $k_0$ such that $$left(fracpqright)^k_0leq
> 2cdot
> left(1-fracq2right)quadquadquadmboxandquadquadquad
> p^k-1.99^k> 1.$$



We will prove that for every $kgeq k_0$ if $|S|inleft(1.99^k,
> p^kright)$
then



there is a strategy for the player $A$ to select sets $P_1$, $P_2$,
$dots$ (based on sets $D_1$, $D_2$, $dots$ provided by $B$) such
that for each $j$ the following relation holds: $P_jcup
> P_j+1cupcdotscup P_j+k=S.$



Assuming that $S=1,2,dots, N$, the player $A$ will maintain the
following sequence of $N$-tuples:
$(mathbfx)_j=0^infty=left(x_1^j, x_2^j, dots, x_N^jright)$.
Initially we set $x_1^0=x_2^0=cdots =x_N^0=1$. After the set $P_j$ is
selected then we define $mathbf x^j+1$ based on $mathbf x^j$ as
follows:



$$x_i^j+1=left{beginarrayrl 1,&mbox if iin P_j \ qcdot
> x_i^j, &mbox if inotin P_j.endarrayright.$$



The player $A$ can keep $B$ from winning if $x_i^jleq q^k$ for each
pair $(i,j)$. For a sequence $mathbf x$, let us define $T(mathbf
> x)=sum_i=1^N x_i$
. It suffices for player $A$ to make sure that
$Tleft(mathbf x^jright)leq q^k$ for each $j$.



Notice that $Tleft(mathbf x^0right)=Nleq p^k < q^k$.



We will now prove that given $mathbf x^j$ such that $Tleft(mathbf
> x^jright)leq q^k$
, and a set $D_j+1$ the player $A$ can choose
$P_j+1inleftD_j+1,D_j+1^Cright$ such that $Tleft(mathbf
> x^j+1right)leq q^k$
.



Let $mathbf y$ be the sequence that would be obtained if
$P_j+1=D_j+1$, and let $mathbf z$ be the sequence that would be
obtained if $P_j+1=D_j+1^C$. Then we have



$$Tleft(mathbf yright)=sum_iin D_j+1^C
> qx_i^j+left|D_j+1right|$$



$$Tleft(mathbf zright)=sum_iin D_j+1
> qx_i^j+left|D_j+1^Cright|.$$



Summing up the previous two equalities gives:



$$Tleft(mathbf yright)+Tleft(mathbf zright)= qcdot
> Tleft(mathbf x^jright)+ Nleq q^k+1+ p^k, mbox hence$$



$$minleftTleft(mathbf yright),Tleft(mathbf
> zright)rightleq fracq2cdot q^k+fracp^k2leq q^k,$$



because of our choice of $k_0$.




I welcome any copyright-motivated edits to this answer before I have time to make my own.






share|cite|improve this answer









$endgroup$












    Your Answer





    StackExchange.ifUsing("editor", function ()
    return StackExchange.using("mathjaxEditing", function ()
    StackExchange.MarkdownEditor.creationCallbacks.add(function (editor, postfix)
    StackExchange.mathjaxEditing.prepareWmdForMathJax(editor, postfix, [["$", "$"], ["\\(","\\)"]]);
    );
    );
    , "mathjax-editing");

    StackExchange.ready(function()
    var channelOptions =
    tags: "".split(" "),
    id: "69"
    ;
    initTagRenderer("".split(" "), "".split(" "), channelOptions);

    StackExchange.using("externalEditor", function()
    // Have to fire editor after snippets, if snippets enabled
    if (StackExchange.settings.snippets.snippetsEnabled)
    StackExchange.using("snippets", function()
    createEditor();
    );

    else
    createEditor();

    );

    function createEditor()
    StackExchange.prepareEditor(
    heartbeatType: 'answer',
    autoActivateHeartbeat: false,
    convertImagesToLinks: true,
    noModals: true,
    showLowRepImageUploadWarning: true,
    reputationToPostImages: 10,
    bindNavPrevention: true,
    postfix: "",
    imageUploader:
    brandingHtml: "Powered by u003ca class="icon-imgur-white" href="https://imgur.com/"u003eu003c/au003e",
    contentPolicyHtml: "User contributions licensed under u003ca href="https://creativecommons.org/licenses/by-sa/3.0/"u003ecc by-sa 3.0 with attribution requiredu003c/au003e u003ca href="https://stackoverflow.com/legal/content-policy"u003e(content policy)u003c/au003e",
    allowUrls: true
    ,
    noCode: true, onDemand: true,
    discardSelector: ".discard-answer"
    ,immediatelyShowMarkdownHelp:true
    );



    );













    draft saved

    draft discarded


















    StackExchange.ready(
    function ()
    StackExchange.openid.initPostLogin('.new-post-login', 'https%3a%2f%2fmath.stackexchange.com%2fquestions%2f3151524%2fcombinatorics-question-from-imo-2012%23new-answer', 'question_page');

    );

    Post as a guest















    Required, but never shown

























    1 Answer
    1






    active

    oldest

    votes








    1 Answer
    1






    active

    oldest

    votes









    active

    oldest

    votes






    active

    oldest

    votes









    1












    $begingroup$

    All but the youngest IMO problems have solutions on imomath.com. I've reproduced verbatim from here:




    The game can be reformulated in an equivalent one: The player $A$
    chooses an element $x$ from the set $S$ (with $|S|=N$) and the player
    $B$ asks the sequence of questions. The $j$-th question consists of
    $B$ choosing a set $D_jsubseteq S$ and player $A$ selecting a set
    $P_jinleft Q_j,Q_j^Cright$. The player $A$ has to make sure
    that for every $jgeq 1$ the following relation holds: $xin P_jcup
    > P_j+1cupcdots cup P_j+k$
    .



    The player $B$ wins if after a finite number of steps he can choose a
    set $X$ with $|X|leq n$ such that $xin X$.



    (a) It suffices to prove that if $Ngeq 2^k+1$ then the player $B$ can
    determine a set $S^primesubseteq S$ with $|S^prime|leq N-1$
    such that $xin S^prime$.



    Assume that $Ngeq 2^n+1$.



    In the first move $B$ selects any set $D_1subseteq S$ such that
    $|D_1|geq 2^k-1$ and $|D_1^C|geq 2^k-1$. After receiving the set
    $P_1$ from $A$, $B$ makes the second move. The player $B$ selects a
    set $D_2subseteq S$ such that $|D_2cap P_1^C|geq 2^k-2$ and
    $|D_2^Ccap P_1^C|geq 2^k-2$. The player $B$ continues this way: in
    the move $j$ he/she chooses a set $D_j$ such that $|D_jcap P_j^C|geq
    > 2^k-j$
    and $|D_j^Ccap P_j^C|geq 2^k-j$.



    In this way the player $B$ has obtained the sets $P_1$, $P_2$,
    $dots$, $P_k$ such that $left(P_1cup cdots cup P_kright)^Cgeq
    > 1$
    . Then $B$ chooses the set $D_k+1$ to be a singleton containing
    any element outside of $P_1cupcdots cup P_k$. There are two cases
    now:



    • The player $A$ selects $P_k+1=D_k+1^C$. Then $B$ can take $S^prime=Ssetminus D_k+1$ and the statement is proved.

    • The player $A$ selects $P_k+1=D_k+1$. Now the player $B$ repeats the previous procedure on the set $S_1=Ssetminus D_k+1$
      to obtain the sequence of sets $P_k+2$, $P_k+3$, $dots$,
      $P_2k+1$. The following inequality holds: $$left|S_1setminus
      > left(P_k+2cdots P_2k+1right)right|geq 1,$$

      since $|S_1|geq 2^k$. However, now we have $$left|left(P_k+1cup P_k+2cupcdotscup
      > P_2k+1right)^Cright|geq 1,$$
      and we may take
      $S^prime=P_k+1cup cdots cup P_2k+1$.

    (b) Let $p$ and $q$ be two positive integers such that $1.99< p< q<
    > 2$
    . Let us choose $k_0$ such that $$left(fracpqright)^k_0leq
    > 2cdot
    > left(1-fracq2right)quadquadquadmboxandquadquadquad
    > p^k-1.99^k> 1.$$



    We will prove that for every $kgeq k_0$ if $|S|inleft(1.99^k,
    > p^kright)$
    then



    there is a strategy for the player $A$ to select sets $P_1$, $P_2$,
    $dots$ (based on sets $D_1$, $D_2$, $dots$ provided by $B$) such
    that for each $j$ the following relation holds: $P_jcup
    > P_j+1cupcdotscup P_j+k=S.$



    Assuming that $S=1,2,dots, N$, the player $A$ will maintain the
    following sequence of $N$-tuples:
    $(mathbfx)_j=0^infty=left(x_1^j, x_2^j, dots, x_N^jright)$.
    Initially we set $x_1^0=x_2^0=cdots =x_N^0=1$. After the set $P_j$ is
    selected then we define $mathbf x^j+1$ based on $mathbf x^j$ as
    follows:



    $$x_i^j+1=left{beginarrayrl 1,&mbox if iin P_j \ qcdot
    > x_i^j, &mbox if inotin P_j.endarrayright.$$



    The player $A$ can keep $B$ from winning if $x_i^jleq q^k$ for each
    pair $(i,j)$. For a sequence $mathbf x$, let us define $T(mathbf
    > x)=sum_i=1^N x_i$
    . It suffices for player $A$ to make sure that
    $Tleft(mathbf x^jright)leq q^k$ for each $j$.



    Notice that $Tleft(mathbf x^0right)=Nleq p^k < q^k$.



    We will now prove that given $mathbf x^j$ such that $Tleft(mathbf
    > x^jright)leq q^k$
    , and a set $D_j+1$ the player $A$ can choose
    $P_j+1inleftD_j+1,D_j+1^Cright$ such that $Tleft(mathbf
    > x^j+1right)leq q^k$
    .



    Let $mathbf y$ be the sequence that would be obtained if
    $P_j+1=D_j+1$, and let $mathbf z$ be the sequence that would be
    obtained if $P_j+1=D_j+1^C$. Then we have



    $$Tleft(mathbf yright)=sum_iin D_j+1^C
    > qx_i^j+left|D_j+1right|$$



    $$Tleft(mathbf zright)=sum_iin D_j+1
    > qx_i^j+left|D_j+1^Cright|.$$



    Summing up the previous two equalities gives:



    $$Tleft(mathbf yright)+Tleft(mathbf zright)= qcdot
    > Tleft(mathbf x^jright)+ Nleq q^k+1+ p^k, mbox hence$$



    $$minleftTleft(mathbf yright),Tleft(mathbf
    > zright)rightleq fracq2cdot q^k+fracp^k2leq q^k,$$



    because of our choice of $k_0$.




    I welcome any copyright-motivated edits to this answer before I have time to make my own.






    share|cite|improve this answer









    $endgroup$

















      1












      $begingroup$

      All but the youngest IMO problems have solutions on imomath.com. I've reproduced verbatim from here:




      The game can be reformulated in an equivalent one: The player $A$
      chooses an element $x$ from the set $S$ (with $|S|=N$) and the player
      $B$ asks the sequence of questions. The $j$-th question consists of
      $B$ choosing a set $D_jsubseteq S$ and player $A$ selecting a set
      $P_jinleft Q_j,Q_j^Cright$. The player $A$ has to make sure
      that for every $jgeq 1$ the following relation holds: $xin P_jcup
      > P_j+1cupcdots cup P_j+k$
      .



      The player $B$ wins if after a finite number of steps he can choose a
      set $X$ with $|X|leq n$ such that $xin X$.



      (a) It suffices to prove that if $Ngeq 2^k+1$ then the player $B$ can
      determine a set $S^primesubseteq S$ with $|S^prime|leq N-1$
      such that $xin S^prime$.



      Assume that $Ngeq 2^n+1$.



      In the first move $B$ selects any set $D_1subseteq S$ such that
      $|D_1|geq 2^k-1$ and $|D_1^C|geq 2^k-1$. After receiving the set
      $P_1$ from $A$, $B$ makes the second move. The player $B$ selects a
      set $D_2subseteq S$ such that $|D_2cap P_1^C|geq 2^k-2$ and
      $|D_2^Ccap P_1^C|geq 2^k-2$. The player $B$ continues this way: in
      the move $j$ he/she chooses a set $D_j$ such that $|D_jcap P_j^C|geq
      > 2^k-j$
      and $|D_j^Ccap P_j^C|geq 2^k-j$.



      In this way the player $B$ has obtained the sets $P_1$, $P_2$,
      $dots$, $P_k$ such that $left(P_1cup cdots cup P_kright)^Cgeq
      > 1$
      . Then $B$ chooses the set $D_k+1$ to be a singleton containing
      any element outside of $P_1cupcdots cup P_k$. There are two cases
      now:



      • The player $A$ selects $P_k+1=D_k+1^C$. Then $B$ can take $S^prime=Ssetminus D_k+1$ and the statement is proved.

      • The player $A$ selects $P_k+1=D_k+1$. Now the player $B$ repeats the previous procedure on the set $S_1=Ssetminus D_k+1$
        to obtain the sequence of sets $P_k+2$, $P_k+3$, $dots$,
        $P_2k+1$. The following inequality holds: $$left|S_1setminus
        > left(P_k+2cdots P_2k+1right)right|geq 1,$$

        since $|S_1|geq 2^k$. However, now we have $$left|left(P_k+1cup P_k+2cupcdotscup
        > P_2k+1right)^Cright|geq 1,$$
        and we may take
        $S^prime=P_k+1cup cdots cup P_2k+1$.

      (b) Let $p$ and $q$ be two positive integers such that $1.99< p< q<
      > 2$
      . Let us choose $k_0$ such that $$left(fracpqright)^k_0leq
      > 2cdot
      > left(1-fracq2right)quadquadquadmboxandquadquadquad
      > p^k-1.99^k> 1.$$



      We will prove that for every $kgeq k_0$ if $|S|inleft(1.99^k,
      > p^kright)$
      then



      there is a strategy for the player $A$ to select sets $P_1$, $P_2$,
      $dots$ (based on sets $D_1$, $D_2$, $dots$ provided by $B$) such
      that for each $j$ the following relation holds: $P_jcup
      > P_j+1cupcdotscup P_j+k=S.$



      Assuming that $S=1,2,dots, N$, the player $A$ will maintain the
      following sequence of $N$-tuples:
      $(mathbfx)_j=0^infty=left(x_1^j, x_2^j, dots, x_N^jright)$.
      Initially we set $x_1^0=x_2^0=cdots =x_N^0=1$. After the set $P_j$ is
      selected then we define $mathbf x^j+1$ based on $mathbf x^j$ as
      follows:



      $$x_i^j+1=left{beginarrayrl 1,&mbox if iin P_j \ qcdot
      > x_i^j, &mbox if inotin P_j.endarrayright.$$



      The player $A$ can keep $B$ from winning if $x_i^jleq q^k$ for each
      pair $(i,j)$. For a sequence $mathbf x$, let us define $T(mathbf
      > x)=sum_i=1^N x_i$
      . It suffices for player $A$ to make sure that
      $Tleft(mathbf x^jright)leq q^k$ for each $j$.



      Notice that $Tleft(mathbf x^0right)=Nleq p^k < q^k$.



      We will now prove that given $mathbf x^j$ such that $Tleft(mathbf
      > x^jright)leq q^k$
      , and a set $D_j+1$ the player $A$ can choose
      $P_j+1inleftD_j+1,D_j+1^Cright$ such that $Tleft(mathbf
      > x^j+1right)leq q^k$
      .



      Let $mathbf y$ be the sequence that would be obtained if
      $P_j+1=D_j+1$, and let $mathbf z$ be the sequence that would be
      obtained if $P_j+1=D_j+1^C$. Then we have



      $$Tleft(mathbf yright)=sum_iin D_j+1^C
      > qx_i^j+left|D_j+1right|$$



      $$Tleft(mathbf zright)=sum_iin D_j+1
      > qx_i^j+left|D_j+1^Cright|.$$



      Summing up the previous two equalities gives:



      $$Tleft(mathbf yright)+Tleft(mathbf zright)= qcdot
      > Tleft(mathbf x^jright)+ Nleq q^k+1+ p^k, mbox hence$$



      $$minleftTleft(mathbf yright),Tleft(mathbf
      > zright)rightleq fracq2cdot q^k+fracp^k2leq q^k,$$



      because of our choice of $k_0$.




      I welcome any copyright-motivated edits to this answer before I have time to make my own.






      share|cite|improve this answer









      $endgroup$















        1












        1








        1





        $begingroup$

        All but the youngest IMO problems have solutions on imomath.com. I've reproduced verbatim from here:




        The game can be reformulated in an equivalent one: The player $A$
        chooses an element $x$ from the set $S$ (with $|S|=N$) and the player
        $B$ asks the sequence of questions. The $j$-th question consists of
        $B$ choosing a set $D_jsubseteq S$ and player $A$ selecting a set
        $P_jinleft Q_j,Q_j^Cright$. The player $A$ has to make sure
        that for every $jgeq 1$ the following relation holds: $xin P_jcup
        > P_j+1cupcdots cup P_j+k$
        .



        The player $B$ wins if after a finite number of steps he can choose a
        set $X$ with $|X|leq n$ such that $xin X$.



        (a) It suffices to prove that if $Ngeq 2^k+1$ then the player $B$ can
        determine a set $S^primesubseteq S$ with $|S^prime|leq N-1$
        such that $xin S^prime$.



        Assume that $Ngeq 2^n+1$.



        In the first move $B$ selects any set $D_1subseteq S$ such that
        $|D_1|geq 2^k-1$ and $|D_1^C|geq 2^k-1$. After receiving the set
        $P_1$ from $A$, $B$ makes the second move. The player $B$ selects a
        set $D_2subseteq S$ such that $|D_2cap P_1^C|geq 2^k-2$ and
        $|D_2^Ccap P_1^C|geq 2^k-2$. The player $B$ continues this way: in
        the move $j$ he/she chooses a set $D_j$ such that $|D_jcap P_j^C|geq
        > 2^k-j$
        and $|D_j^Ccap P_j^C|geq 2^k-j$.



        In this way the player $B$ has obtained the sets $P_1$, $P_2$,
        $dots$, $P_k$ such that $left(P_1cup cdots cup P_kright)^Cgeq
        > 1$
        . Then $B$ chooses the set $D_k+1$ to be a singleton containing
        any element outside of $P_1cupcdots cup P_k$. There are two cases
        now:



        • The player $A$ selects $P_k+1=D_k+1^C$. Then $B$ can take $S^prime=Ssetminus D_k+1$ and the statement is proved.

        • The player $A$ selects $P_k+1=D_k+1$. Now the player $B$ repeats the previous procedure on the set $S_1=Ssetminus D_k+1$
          to obtain the sequence of sets $P_k+2$, $P_k+3$, $dots$,
          $P_2k+1$. The following inequality holds: $$left|S_1setminus
          > left(P_k+2cdots P_2k+1right)right|geq 1,$$

          since $|S_1|geq 2^k$. However, now we have $$left|left(P_k+1cup P_k+2cupcdotscup
          > P_2k+1right)^Cright|geq 1,$$
          and we may take
          $S^prime=P_k+1cup cdots cup P_2k+1$.

        (b) Let $p$ and $q$ be two positive integers such that $1.99< p< q<
        > 2$
        . Let us choose $k_0$ such that $$left(fracpqright)^k_0leq
        > 2cdot
        > left(1-fracq2right)quadquadquadmboxandquadquadquad
        > p^k-1.99^k> 1.$$



        We will prove that for every $kgeq k_0$ if $|S|inleft(1.99^k,
        > p^kright)$
        then



        there is a strategy for the player $A$ to select sets $P_1$, $P_2$,
        $dots$ (based on sets $D_1$, $D_2$, $dots$ provided by $B$) such
        that for each $j$ the following relation holds: $P_jcup
        > P_j+1cupcdotscup P_j+k=S.$



        Assuming that $S=1,2,dots, N$, the player $A$ will maintain the
        following sequence of $N$-tuples:
        $(mathbfx)_j=0^infty=left(x_1^j, x_2^j, dots, x_N^jright)$.
        Initially we set $x_1^0=x_2^0=cdots =x_N^0=1$. After the set $P_j$ is
        selected then we define $mathbf x^j+1$ based on $mathbf x^j$ as
        follows:



        $$x_i^j+1=left{beginarrayrl 1,&mbox if iin P_j \ qcdot
        > x_i^j, &mbox if inotin P_j.endarrayright.$$



        The player $A$ can keep $B$ from winning if $x_i^jleq q^k$ for each
        pair $(i,j)$. For a sequence $mathbf x$, let us define $T(mathbf
        > x)=sum_i=1^N x_i$
        . It suffices for player $A$ to make sure that
        $Tleft(mathbf x^jright)leq q^k$ for each $j$.



        Notice that $Tleft(mathbf x^0right)=Nleq p^k < q^k$.



        We will now prove that given $mathbf x^j$ such that $Tleft(mathbf
        > x^jright)leq q^k$
        , and a set $D_j+1$ the player $A$ can choose
        $P_j+1inleftD_j+1,D_j+1^Cright$ such that $Tleft(mathbf
        > x^j+1right)leq q^k$
        .



        Let $mathbf y$ be the sequence that would be obtained if
        $P_j+1=D_j+1$, and let $mathbf z$ be the sequence that would be
        obtained if $P_j+1=D_j+1^C$. Then we have



        $$Tleft(mathbf yright)=sum_iin D_j+1^C
        > qx_i^j+left|D_j+1right|$$



        $$Tleft(mathbf zright)=sum_iin D_j+1
        > qx_i^j+left|D_j+1^Cright|.$$



        Summing up the previous two equalities gives:



        $$Tleft(mathbf yright)+Tleft(mathbf zright)= qcdot
        > Tleft(mathbf x^jright)+ Nleq q^k+1+ p^k, mbox hence$$



        $$minleftTleft(mathbf yright),Tleft(mathbf
        > zright)rightleq fracq2cdot q^k+fracp^k2leq q^k,$$



        because of our choice of $k_0$.




        I welcome any copyright-motivated edits to this answer before I have time to make my own.






        share|cite|improve this answer









        $endgroup$



        All but the youngest IMO problems have solutions on imomath.com. I've reproduced verbatim from here:




        The game can be reformulated in an equivalent one: The player $A$
        chooses an element $x$ from the set $S$ (with $|S|=N$) and the player
        $B$ asks the sequence of questions. The $j$-th question consists of
        $B$ choosing a set $D_jsubseteq S$ and player $A$ selecting a set
        $P_jinleft Q_j,Q_j^Cright$. The player $A$ has to make sure
        that for every $jgeq 1$ the following relation holds: $xin P_jcup
        > P_j+1cupcdots cup P_j+k$
        .



        The player $B$ wins if after a finite number of steps he can choose a
        set $X$ with $|X|leq n$ such that $xin X$.



        (a) It suffices to prove that if $Ngeq 2^k+1$ then the player $B$ can
        determine a set $S^primesubseteq S$ with $|S^prime|leq N-1$
        such that $xin S^prime$.



        Assume that $Ngeq 2^n+1$.



        In the first move $B$ selects any set $D_1subseteq S$ such that
        $|D_1|geq 2^k-1$ and $|D_1^C|geq 2^k-1$. After receiving the set
        $P_1$ from $A$, $B$ makes the second move. The player $B$ selects a
        set $D_2subseteq S$ such that $|D_2cap P_1^C|geq 2^k-2$ and
        $|D_2^Ccap P_1^C|geq 2^k-2$. The player $B$ continues this way: in
        the move $j$ he/she chooses a set $D_j$ such that $|D_jcap P_j^C|geq
        > 2^k-j$
        and $|D_j^Ccap P_j^C|geq 2^k-j$.



        In this way the player $B$ has obtained the sets $P_1$, $P_2$,
        $dots$, $P_k$ such that $left(P_1cup cdots cup P_kright)^Cgeq
        > 1$
        . Then $B$ chooses the set $D_k+1$ to be a singleton containing
        any element outside of $P_1cupcdots cup P_k$. There are two cases
        now:



        • The player $A$ selects $P_k+1=D_k+1^C$. Then $B$ can take $S^prime=Ssetminus D_k+1$ and the statement is proved.

        • The player $A$ selects $P_k+1=D_k+1$. Now the player $B$ repeats the previous procedure on the set $S_1=Ssetminus D_k+1$
          to obtain the sequence of sets $P_k+2$, $P_k+3$, $dots$,
          $P_2k+1$. The following inequality holds: $$left|S_1setminus
          > left(P_k+2cdots P_2k+1right)right|geq 1,$$

          since $|S_1|geq 2^k$. However, now we have $$left|left(P_k+1cup P_k+2cupcdotscup
          > P_2k+1right)^Cright|geq 1,$$
          and we may take
          $S^prime=P_k+1cup cdots cup P_2k+1$.

        (b) Let $p$ and $q$ be two positive integers such that $1.99< p< q<
        > 2$
        . Let us choose $k_0$ such that $$left(fracpqright)^k_0leq
        > 2cdot
        > left(1-fracq2right)quadquadquadmboxandquadquadquad
        > p^k-1.99^k> 1.$$



        We will prove that for every $kgeq k_0$ if $|S|inleft(1.99^k,
        > p^kright)$
        then



        there is a strategy for the player $A$ to select sets $P_1$, $P_2$,
        $dots$ (based on sets $D_1$, $D_2$, $dots$ provided by $B$) such
        that for each $j$ the following relation holds: $P_jcup
        > P_j+1cupcdotscup P_j+k=S.$



        Assuming that $S=1,2,dots, N$, the player $A$ will maintain the
        following sequence of $N$-tuples:
        $(mathbfx)_j=0^infty=left(x_1^j, x_2^j, dots, x_N^jright)$.
        Initially we set $x_1^0=x_2^0=cdots =x_N^0=1$. After the set $P_j$ is
        selected then we define $mathbf x^j+1$ based on $mathbf x^j$ as
        follows:



        $$x_i^j+1=left{beginarrayrl 1,&mbox if iin P_j \ qcdot
        > x_i^j, &mbox if inotin P_j.endarrayright.$$



        The player $A$ can keep $B$ from winning if $x_i^jleq q^k$ for each
        pair $(i,j)$. For a sequence $mathbf x$, let us define $T(mathbf
        > x)=sum_i=1^N x_i$
        . It suffices for player $A$ to make sure that
        $Tleft(mathbf x^jright)leq q^k$ for each $j$.



        Notice that $Tleft(mathbf x^0right)=Nleq p^k < q^k$.



        We will now prove that given $mathbf x^j$ such that $Tleft(mathbf
        > x^jright)leq q^k$
        , and a set $D_j+1$ the player $A$ can choose
        $P_j+1inleftD_j+1,D_j+1^Cright$ such that $Tleft(mathbf
        > x^j+1right)leq q^k$
        .



        Let $mathbf y$ be the sequence that would be obtained if
        $P_j+1=D_j+1$, and let $mathbf z$ be the sequence that would be
        obtained if $P_j+1=D_j+1^C$. Then we have



        $$Tleft(mathbf yright)=sum_iin D_j+1^C
        > qx_i^j+left|D_j+1right|$$



        $$Tleft(mathbf zright)=sum_iin D_j+1
        > qx_i^j+left|D_j+1^Cright|.$$



        Summing up the previous two equalities gives:



        $$Tleft(mathbf yright)+Tleft(mathbf zright)= qcdot
        > Tleft(mathbf x^jright)+ Nleq q^k+1+ p^k, mbox hence$$



        $$minleftTleft(mathbf yright),Tleft(mathbf
        > zright)rightleq fracq2cdot q^k+fracp^k2leq q^k,$$



        because of our choice of $k_0$.




        I welcome any copyright-motivated edits to this answer before I have time to make my own.







        share|cite|improve this answer












        share|cite|improve this answer



        share|cite|improve this answer










        answered Mar 17 at 16:44









        J.G.J.G.

        32.1k23250




        32.1k23250



























            draft saved

            draft discarded
















































            Thanks for contributing an answer to Mathematics Stack Exchange!


            • Please be sure to answer the question. Provide details and share your research!

            But avoid


            • Asking for help, clarification, or responding to other answers.

            • Making statements based on opinion; back them up with references or personal experience.

            Use MathJax to format equations. MathJax reference.


            To learn more, see our tips on writing great answers.




            draft saved


            draft discarded














            StackExchange.ready(
            function ()
            StackExchange.openid.initPostLogin('.new-post-login', 'https%3a%2f%2fmath.stackexchange.com%2fquestions%2f3151524%2fcombinatorics-question-from-imo-2012%23new-answer', 'question_page');

            );

            Post as a guest















            Required, but never shown





















































            Required, but never shown














            Required, but never shown












            Required, but never shown







            Required, but never shown

































            Required, but never shown














            Required, but never shown












            Required, but never shown







            Required, but never shown







            Popular posts from this blog

            Lowndes Grove History Architecture References Navigation menu32°48′6″N 79°57′58″W / 32.80167°N 79.96611°W / 32.80167; -79.9661132°48′6″N 79°57′58″W / 32.80167°N 79.96611°W / 32.80167; -79.9661178002500"National Register Information System"Historic houses of South Carolina"Lowndes Grove""+32° 48' 6.00", −79° 57' 58.00""Lowndes Grove, Charleston County (260 St. Margaret St., Charleston)""Lowndes Grove"The Charleston ExpositionIt Happened in South Carolina"Lowndes Grove (House), Saint Margaret Street & Sixth Avenue, Charleston, Charleston County, SC(Photographs)"Plantations of the Carolina Low Countrye

            random experiment with two different functions on unit interval Announcing the arrival of Valued Associate #679: Cesar Manara Planned maintenance scheduled April 23, 2019 at 00:00UTC (8:00pm US/Eastern)Random variable and probability space notionsRandom Walk with EdgesFinding functions where the increase over a random interval is Poisson distributedNumber of days until dayCan an observed event in fact be of zero probability?Unit random processmodels of coins and uniform distributionHow to get the number of successes given $n$ trials , probability $P$ and a random variable $X$Absorbing Markov chain in a computer. Is “almost every” turned into always convergence in computer executions?Stopped random walk is not uniformly integrable

            How should I support this large drywall patch? Planned maintenance scheduled April 23, 2019 at 00:00UTC (8:00pm US/Eastern) Announcing the arrival of Valued Associate #679: Cesar Manara Unicorn Meta Zoo #1: Why another podcast?How do I cover large gaps in drywall?How do I keep drywall around a patch from crumbling?Can I glue a second layer of drywall?How to patch long strip on drywall?Large drywall patch: how to avoid bulging seams?Drywall Mesh Patch vs. Bulge? To remove or not to remove?How to fix this drywall job?Prep drywall before backsplashWhat's the best way to fix this horrible drywall patch job?Drywall patching using 3M Patch Plus Primer